Jump to content

Symmetry breaking Lagrangian


Genady

Recommended Posts

This is a multi-step exercise. It would be very helpful if somebody could check my step(s) as I go. @joigus, I'm sure it is a child play for you.

I'd like to make sure that I've derived correctly the equation of motion for this Lagrangian:

\[\mathcal L=- \frac 1 2 \phi \Box \phi + \frac 1 2 m^2 \phi^2 - \frac {\lambda} {4!} \phi^4\]

The EL equation:

\[\frac {\partial \mathcal L} {\partial \phi} + \Box \frac {\partial \mathcal L} {\partial (\Box \phi)} = 0\]

The equation of motion:

\[\Box \phi - \frac 1 2 m^2 + \frac {\lambda} {3!} \phi^3 = 0\]

 

How is it?

P.S. As edit LaTex does not work, I add a typo correction here. The equation of motion is rather

\[\Box \phi - m^2 \phi + \frac {\lambda} {3!} \phi^3 = 0\]

Link to comment
Share on other sites

12 hours ago, Genady said:

This is a multi-step exercise. It would be very helpful if somebody could check my step(s) as I go. @joigus, I'm sure it is a child play for you.

I'd like to make sure that I've derived correctly the equation of motion for this Lagrangian:

 

L=12ϕϕ+12m2ϕ2λ4!ϕ4

 

The EL equation:

 

Lϕ+L(ϕ)=0

 

The equation of motion:

 

ϕ12m2+λ3!ϕ3=0

 

 

How is it?

P.S. As edit LaTex does not work, I add a typo correction here. The equation of motion is rather

 

ϕm2ϕ+λ3!ϕ3=0

 

Your eq. of motion (once corrected) looks fine. Most people prefer to write (1/2)(grad)phi(grad)phi instead of -(1/2)phi(grad)2phi, but they differ in just a total divergence, so they are equivalent (lead to the same equations of motion).

By grad2 I mean the D'Alembert operator. I'll check in more detail later, I would have to do it in my head now and I could miss a sign. This is a famous equation.

Link to comment
Share on other sites

Q: How many constants \(c\) are there so that \(\phi(x)=c\) is a solution to the equation of motion?

A: Three. \(c=0\) and two solutions for \(c^2= \frac {3!} {\lambda} m^2\)

Q: Which solution has the lowest energy (the ground state)?

A: The potential energy from the Lagrangian is \[\frac {\lambda} {4!} \phi^4 - \frac 1 2 m^2 \phi^2\]It is the lowest for the non-zero \(c\): \(- \frac{3!m^4} {4 \lambda}\).

Link to comment
Share on other sites

Your derivation of the equation of motion from the given Lagrangian appears to be mostly correct, but there's a small mistake in your final result. Let's go through it step by step:

Given Lagrangian:

4L=21ϕϕ+21m2ϕ24!λϕ4

The Euler-Lagrange equation is:

ϕL+((ϕ)L)=0

Taking the derivatives with respect to ϕ and 3ϕL=ϕ+m2ϕ3!λϕ3(ϕ)L=21

Now, plugging these derivatives into the Euler-Lagrange equation: ϕ+m2ϕ3!λϕ3+(21)=0

Simplifying: ϕ+m2ϕ3!λϕ3+21=0

Rearranging terms: ϕm2ϕ+3!λϕ3=0

So, the corrected equation of motion should be: ϕm2ϕ+3!λϕ3=0

This is the correct equation of motion derived from the given Lagrangian.

If you want to know more about lagrang theorem,I will suggest you to visit link removed once.

Link to comment
Share on other sites

Ok. Yes, @RobertSmart is right. There is a little mistake in the constants. Let me display my calculation in detail, because his Latex seems to have been messed up by the compiling engine or whatever and I seem to find a small discrepancy with him.

Your Lagrangian,

\[ \mathscr{\mathcal{L}}=-\frac{1}{2}\phi\Box\phi+\frac{1}{2}m^{2}\phi^{2}-\frac{\lambda}{4!}\phi^{4} \]

I prefer to write with an index notation, which is more convenient for variational derivatives:

\[ \mathscr{\mathcal{L}}=-\frac{1}{2}\phi\left.\phi^{,\mu}\right._{,\mu}+\frac{1}{2}m^{2}\phi^{2}-\frac{\lambda}{4!}\phi^{4} \]

As we have no dependence on first order derivatives,

\[ \frac{\partial\mathscr{\mathcal{L}}}{\partial\phi_{,\mu}}=0 \]

we get as the only Euler-Lagrange equation,

\[ \frac{\partial\mathscr{\mathcal{L}}}{\partial\phi}=-\frac{1}{2}\left.\phi^{,\mu}\right._{,\mu}+m^{2}\phi-\frac{\lambda}{3!}\phi^{3}=0 \]

Or,

\[ -\frac{1}{2}\Box\phi+m^{2}\phi-\frac{\lambda}{3!}\phi^{3}=0 \]

Or a bit more streamlined,

\[ \Box\phi-2m^{2}\phi+\frac{\lambda}{3}\phi^{3}=0 \]

Sorry I didn't get around to it sooner.

Paraphrasing Sir Humphrey Appleby:

Is that finally final?

I hope so.

PS: BTW, this is a simplified symmetry-breaking Lagrangian. The real thing in the SM is a complex SU(2)-symmetric multiplet \( \left(\phi_{1},\phi_{2},\phi_{3},\phi_{4}\right) \).

Edited by joigus
Latex editing
Link to comment
Share on other sites

Unfortunately, I can't read this post:

1 hour ago, RobertSmart said:

Given Lagrangian:

4L=21ϕϕ+21m2ϕ24!λϕ4

The Euler-Lagrange equation is:

ϕL+((ϕ)L)=0

and I don't know how his result is different from mine, but it seems that his EL equation is the same as mine,

On 2/27/2024 at 6:36 PM, Genady said:

The EL equation:

image.png.23a3d9c9c56842990394a799aaa24997.png

<<<<<

which is different from

36 minutes ago, joigus said:

we get as the only Euler-Lagrange equation,

image.png.487bfffa94feba16627f51ec14469b48.png

<<<<<<<

I disagree with the latter. We need to use the generalized EL equation, which I have already derived in this exercise:

image.png.ee6a920159763cf92a405c29baa04058.png

and got the answer compatible with this:

image.png.c69d0ad22534ead4957aea82611cc7e6.png

(Euler–Lagrange equation - Wikipedia)

 

Edited by Genady
Link to comment
Share on other sites

12 minutes ago, Genady said:

Unfortunately, I can't read this post:

and I don't know how his result is different from mine, but it seems that his EL equation is the same as mine,

image.png.23a3d9c9c56842990394a799aaa24997.png

<<<<<

which is different from

image.png.487bfffa94feba16627f51ec14469b48.png

<<<<<<<

I disagree with the latter. We need to use the generalized EL equation, which I have already derived in this exercise:

image.png.ee6a920159763cf92a405c29baa04058.png

and got the answer compatible with this:

image.png.c69d0ad22534ead4957aea82611cc7e6.png

(Euler–Lagrange equation - Wikipedia)

 

Yes, you're right. The unusual writing of the Lagrangian set me off. Sorry. That is indeed the way to generalise to higher-order derivatives. I've proven it many times, but now I had just a couple of minutes and I screwed up. There's just a coefficient difference. 

Later.

Link to comment
Share on other sites

Create an account or sign in to comment

You need to be a member in order to leave a comment

Create an account

Sign up for a new account in our community. It's easy!

Register a new account

Sign in

Already have an account? Sign in here.

Sign In Now
×
×
  • Create New...

Important Information

We have placed cookies on your device to help make this website better. You can adjust your cookie settings, otherwise we'll assume you're okay to continue.